Difference between revisions of "1988 AIME Problems/Problem 11"

m (See also)
(solution)
Line 1: Line 1:
 
== Problem ==
 
== Problem ==
 +
Let <math>w_1, w_2, \dots, w_n</math> be [[complex number]]s.  A line <math>L</math> in the [[complex plane]] is called a mean [[line]] for the [[point]]s <math>w_1, w_2, \dots, w_n</math> if <math>L</math> contains points (complex numbers) <math>z_1, z_2, \dots, z_n</math> such that
 +
<cmath>
 +
\sum_{k = 1}^n (z_k - w_k) = 0.
 +
</cmath>
 +
For the numbers <math>w_1 = 32 + 170i</math>, <math>w_2 = - 7 + 64i</math>, <math>w_3 = - 9 + 200i</math>, <math>w_4 = 1 + 27i</math>, and <math>w_5 = - 14 + 43i</math>, there is a unique mean line with <math>y</math>-intercept 3.  Find the [[slope]] of this mean line.
  
 
== Solution ==
 
== Solution ==
 +
<math>\sum_{k=1}^5 z_k - \sum_{k=1}^5 w_k = 0</math>
 +
 +
<math>\sum_{k=1}^5 z_k = 3 + 504i</math>
 +
 +
Each <math>z_k = x_k + y_ki</math> lies on the complex line <math>y = mx + 3</math>, so we can rewrite this as
 +
 +
<math>\sum_{k=1}^5 z_k = \sum_{k=1}^5 x_k + \sum_{k=1}^n y_ki</math>
 +
 +
<math>3 + 504i = \sum_{k=1}^5 x_k + i \sum_{k=1}^5 (mx_k + 3)</math>
 +
 +
Matching the real parts and the imaginary parts, we get that <math>\sum_{k=1}^5 x_k = 3</math> and <math>\sum_{k=1}^5 (mx_k + 3) = 504</math>. Simplifying the second summation, we find that <math>m\sum_{k=1}^5 x_k = 504 - 3 \cdot 5 = 489</math>, and substituting, the answer is <math>m \cdot 3 = 489 \Longrightarrow m = 163</math>.
  
 
== See also ==
 
== See also ==
* [[1988 AIME Problems]]
+
{{AIME box|year=1988|num-b=10|num-a=12}}
  
{{AIME box|year=1988|num-b=10|num-a=12}}
+
[[Category:Intermediate Complex Numbers Problems]]
 +
[[Category:Intermediate Geometry Problems]]

Revision as of 17:10, 28 September 2007

Problem

Let $w_1, w_2, \dots, w_n$ be complex numbers. A line $L$ in the complex plane is called a mean line for the points $w_1, w_2, \dots, w_n$ if $L$ contains points (complex numbers) $z_1, z_2, \dots, z_n$ such that \[\sum_{k = 1}^n (z_k - w_k) = 0.\] For the numbers $w_1 = 32 + 170i$, $w_2 = - 7 + 64i$, $w_3 = - 9 + 200i$, $w_4 = 1 + 27i$, and $w_5 = - 14 + 43i$, there is a unique mean line with $y$-intercept 3. Find the slope of this mean line.

Solution

$\sum_{k=1}^5 z_k - \sum_{k=1}^5 w_k = 0$

$\sum_{k=1}^5 z_k = 3 + 504i$

Each $z_k = x_k + y_ki$ lies on the complex line $y = mx + 3$, so we can rewrite this as

$\sum_{k=1}^5 z_k = \sum_{k=1}^5 x_k + \sum_{k=1}^n y_ki$

$3 + 504i = \sum_{k=1}^5 x_k + i \sum_{k=1}^5 (mx_k + 3)$

Matching the real parts and the imaginary parts, we get that $\sum_{k=1}^5 x_k = 3$ and $\sum_{k=1}^5 (mx_k + 3) = 504$. Simplifying the second summation, we find that $m\sum_{k=1}^5 x_k = 504 - 3 \cdot 5 = 489$, and substituting, the answer is $m \cdot 3 = 489 \Longrightarrow m = 163$.

See also

1988 AIME (ProblemsAnswer KeyResources)
Preceded by
Problem 10
Followed by
Problem 12
1 2 3 4 5 6 7 8 9 10 11 12 13 14 15
All AIME Problems and Solutions